¿Por qué los puntos de Lagrange L1L1L_1, L2L2L_2 y L3L3L_3 son inestables?

¿Por qué los puntos de Lagrange son L 1 , L 2 & L 3 ¿inestable? Estoy haciendo una presentación de física frente a mi clase y solo estoy confirmando que esto es correcto, y si no lo es, ¿alguien podría dar una explicación?

F = metro v 2 / r y v = 2 π / T

Por lo tanto:

F = 4 π 2 metro r / T 2

F 1 / T 2 mientras r es constante

Así que si r deja de estar en el mismo punto que el punto de Lagrange, entonces el período de la órbita cambiará y el sistema caerá fuera de lugar.

Respuestas (3)

I) OP pregunta (v5):

¿Por qué los puntos de Lagrange son L 1 , L 2 & L 3 ¿inestable?

En realidad, esa es una gran pregunta y no trivial. No se deben sacar conclusiones basadas simplemente en el potencial efectivo V (que consiste en la gravedad y el potencial centrífugo ) solo, porque la fuerza de Coriolis puede ser importante.

Por ejemplo, los puntos máximos globales L 4 & L 5 de V (que ingenuamente serían los últimos lugares para sospechar un punto de equilibrio estable), son a veces estables, cf. por ejemplo, esta publicación de Phys.SE!

II) Analicemos aquí la estabilidad de un punto arbitrario con pequeña velocidad inicial v 0 en el plano orbital 1 R 2 C en presencia de la fuerza de Coriolis. Escojamos coordenadas z C para que la posición inicial esté en el origen z = 0 .

Comencemos por estudiar los puntos críticos/estacionarios , es decir, los puntos de Lagrange. Usamos el siguiente teorema mencionado en la Ref. 1:

Teorema. Dada una arpillera H para un potencial efectivo V : R 2 R en un punto de Lagrange. Dejar Ω Sea la velocidad angular. Las siguientes 3 condiciones son condiciones necesarias y suficientes para que el punto de Lagrange sea estable:

  1. C   :=   det ( H )     0.

  2. B   :=   t r ( H ) + 4 Ω 2     0.

  3. D   :=   B 2 4 C     0.

NB: El teorema ignora los términos de orden superior en V , que podría volverse importante si det ( H ) = 0 .

Se proporciona una prueba del teorema en mi Phys.SE answer here , que también analiza el caso de un máximo local.

  1. Caso de un mínimo local 2 det ( H ) 0 t r ( H ) 0 : Es estable porque se cumplen las 3 condiciones. Para ver la tercera condición, observe que

    (1) D   =   B 2 4 C   =   ( t r ( H ) + 4 Ω 2 ) 2 4 det ( H )   =   t r ( H ) 4 det ( H ) + 8 t r ( H ) Ω 2 + dieciséis Ω 4   =   ( H 11 H 22 ) 2 + 4 H 12 2 + 8 t r ( H ) 0 Ω 2 + dieciséis Ω 4   >   0.

  2. Caso de un punto de silla fuerte det ( H ) < 0 : Este es el caso de los puntos de Lagrange L 1 , L 2 & L 3 , sobre lo que OP pregunta. Es inestable, porque viola la primera condición.

III) Para ganar algo de intuición para las conclusiones anteriores, analicemos también lo que sucede fuera de los puntos de Lagrange.

  1. Caso de un punto no crítico: La fuerza efectiva específica F = V | z = 0 0 es distinto de cero. Luego, en la región donde podemos tratar la fuerza efectiva específica como constante, el EOM con la fuerza específica de Coriolis 2 v × Ω lee 3

    (2) z ¨     2 i Ω z ˙ Coriolis + F   d d t ( Exp { 2 i Ω t } z ˙ )     Exp { 2 i Ω t } F .
    Por lo tanto la velocidad es
    (3) z ˙     F 2 i Ω + ( v 0 F 2 i Ω ) Exp { 2 i Ω t } ,
    y la posición es
    (4) z     F t 2 i Ω deriva + 1 2 i Ω ( F 2 i Ω v 0 ) amplitud compleja ( Exp { 2 i Ω t } círculo 1 )
    (5)     v 0 t + ( F 2 i Ω v 0 ) t 2 2 ( 1 2 i Ω t 3 ) + O ( t 4 ) .
    La partícula de prueba realiza un movimiento circular con una deriva a lo largo de líneas equipotenciales en el plano orbital. (Hacia r i gramo h t si estas buscando d o w norte h i yo yo .) En cualquier caso, la partícula de prueba es inestable en un punto no crítico.

    Fig. 1: Una posible órbita en herradura a lo largo de líneas equipotenciales.

La descripción anterior proporciona información sobre por qué los mínimos locales y (a veces) los máximos locales son estables, mientras que los puntos de silla son siempre inestables. Si la partícula de prueba tiende a desviarse a lo largo de líneas equipotenciales, entonces cerca de los mínimos y máximos locales, termina en pequeños bucles concéntricos, mientras que las líneas equipotenciales siempre apuntan lejos de los puntos de silla.

Fig. 2: Una partícula de prueba tiende a desplazarse a lo largo de líneas equipotenciales debido a la fuerza de Coriolis.

Referencias:

  1. J. Binney & S. Tremaine, Galactic Dynamics, 2ª edición (2008); pag. 181-182.

--

1 También podría haber un movimiento oscilatorio perpendicular al plano orbital, que ignoramos.

2 El análisis anterior es válido para cualquier potencial efectivo V . En el problema restringido de 3 cuerpos, sobre el que OP pregunta, el potencial efectivo V no tiene mínimo local.

3 Aquí elegimos la orientación de la velocidad angular. Ω para ser progrado/en sentido antihorario/sentido positivo de rotación.

Creo que la forma más intuitiva de pensar en los puntos de Lagrange es considerar la energía potencial gravitacional y una superficie que representa el potencial.

Los puntos de Lagrange existen en el espacio alrededor de dos objetos masivos, digamos el Sol y la Tierra. Puede pensar en ambos como si produjeran un pozo de potencial gravitatorio con una forma un poco como un cono o un embudo apuntando hacia abajo pero con lados que forman un perfil curvo. El potencial gravitacional en función de la distancia. r es dado por tu = GRAMO metro / r . Aquí hay una buena foto tomada de Wikipedea 2 . Fuente de la imagen: por AllenMcC. - Trabajo propio, CC BY-SA 3.0, Pozo de Potencial GraviacionalPozo de potencial gravitatorio

Ahora, dado que hay dos cuerpos masivos, hay dos de estos pozos cerca de cada uno, aunque el que está centrado en el cuerpo más masivo será más grande. Puede imaginar que habría un punto de silla entre los dos pozos donde la superficie sería plana. Esto representaría un punto donde las fuerzas de gravedad que tiran hacia cada cuerpo están en equilibrio (no pude encontrar una buena imagen de dos pozos).

La cuestión es que el cuerpo más pequeño (digamos, la Tierra) gira alrededor del cuerpo más grande (el Sol) o, más exactamente, ambos giran alrededor de su baricentro común. Los puntos de Lagrange son estacionarios vistos desde la Tierra, por lo que debemos considerar las cosas desde un marco de referencia que gira con la Tierra. En tal marco de referencia giratorio, tenemos que introducir una fuerza centrífuga ficticia lejos del centro de rotación. La adición de esta fuerza cambia la superficie potencial con los pozos gravitatorios. Dado que la fuerza actúa alejándose del centro, un objeto cerca del centro tendría un potencial más alto que uno más alejado, por lo que el potencial de esta fuerza se vería como un cuenco invertido. Cuando juntas los efectos de la gravedad de ambos cuerpos y la fuerza centrífuga, obtienes una superficie potencial efectiva algo complicada. Aquí'Puntos Lagranianos Equipotenciales

Fuente de la imagen: Por usuario: cmglee - Trabajo propio, CC BY-SA 3.0, Lagranian Points Equipotential

Entonces, los cinco puntos de Lagrange son lugares donde la pendiente de esta superficie potencial es cero. Podrías imaginarte sentado allí y no deslizarte lejos de ellos si tuvieras cuidado. El L 1 , L 2 , L 3 , tienen forma de silla de montar mientras que los L 4 y L 5 los puntos ocurren en los máximos locales. Son como cimas de colinas pero colinas bastante planas.

No obstante, el L 4 y L 5 los puntos no son mínimos locales, entonces, ¿cómo son estables? Bueno, en el marco de referencia giratorio, la fuerza centrífuga no es la única fuerza ficticia que debe introducirse. Una vez que algo comienza a moverse , también experimenta una fuerza de Coriolis perpendicular a su movimiento. Si un satélite se aleja del L 4 o L 5 señala que es esta fuerza de Coriolis la que lo empujará de regreso, si no exactamente al punto de Lagrange, entonces a una órbita alrededor de él.

Me acabo de dar cuenta de que expliqué mal: por qué L4 y L5 son estables. editaré
Un artículo como Stability of Lagrange Points: James Webb Space Telescope brinda un análisis de la estabilidad de los cinco puntos de Lagrange, pero supongo que está más allá del nivel que buscaba el OP. ¿Hay una forma más sencilla de responder a la pregunta original más allá de simplemente decir que el potencial efectivo no es un mínimo local en los puntos en cuestión y que la explicación de la fuerza de Coriolis que usamos para L4 y L5 no funciona para L1, L2 y L3?

A su análisis le faltan dos elementos clave: la fuerza de la gravedad y cómo esa fuerza (y la fuerza centrífuga) varían con r. Necesitas esos. Entra en el marco giratorio alrededor del centro de masa del sistema e ingresa la gravedad de ambos objetos, así como la fuerza centrífuga (está ahí porque estamos en un marco de referencia giratorio). Debe encontrar que todas estas fuerzas suman cero en los puntos L1 y L3. Luego observe que la fuerza apunta lejos de los puntos L1 y L3 si varía un poco. Si no tiene las habilidades matemáticas de cálculo para poder ver esto, simplemente puede decirlo: un objeto justo en L1 o L3 puede moverse con el mismo período orbital que las fuentes de gravedad en órbita, pero cualquier desviación lejos de eso estropea eso. Esto es lo que dijiste, pero nunca especificaste las fuerzas involucradas, por lo que no basta con demostrar que es inestable porque la inestabilidad sí depende de esas fuerzas. Cuánto quieras mostrar eso, o simplemente decirlo, depende de la profundidad de tu presentación.